• Anúncio Global
    Respostas
    Exibições
    Última mensagem

questoes de concurso

Regras do fórum
A classificação destes desafios em fáceis, médios e difíceis, é apenas ilustrativa.
Eventualmente, o que pode ser difícil para a maioria, pode ser fácil para você e vice-versa.

questoes de concurso

Mensagempor karla_paula » Dom Jun 13, 2010 15:06

41. Sejam:
\frac{1^2}{1}+ \frac{2^2}{3}+\frac{3^2}{5}+....\frac{1001^2}{2001}  e   b= \frac{1^2}{3}+\frac{2^2}{5}+\frac{3^2}{7}....\frac{1001^2}{2003}

Qual é o inteiro mais próximo de a – b?
(A) 1001.
(B) 500.
(C) 999.
(D) 1000.
(E) 501
karla_paula
Usuário Ativo
Usuário Ativo
 
Mensagens: 11
Registrado em: Sáb Jun 12, 2010 08:09
Formação Escolar: GRADUAÇÃO
Área/Curso: licenciatura em matematica
Andamento: formado

Re: questoes de concurso

Mensagempor Vininhuu » Seg Jul 12, 2010 15:51

O a = \dfrac{1^2}{1} + \dfrac{2^2}{3} + \dfrac{3^2}{5} + \dfrac{4^2}{7} + \ldots \dfrac{1001^2}{2001}?

É que ali não consta o valor de a, só mostra o "\dfrac{1^2}{1} + \dfrac{2^2}{3} + \dfrac{3^2}{5} + \dfrac{4^2}{7} + \ldots \dfrac{1001^2}{2001}" e o valor de b.
Vininhuu
Novo Usuário
Novo Usuário
 
Mensagens: 8
Registrado em: Qui Jun 10, 2010 17:54
Formação Escolar: ENSINO MÉDIO
Andamento: cursando

Re: questoes de concurso

Mensagempor MarceloFantini » Ter Jul 13, 2010 16:26

Quando tivermos 3 parcelas: (\frac{1^2}{1} + \frac{2^2}{3} + \frac{3^2}{5}) - (\frac{1^2}{3} + \frac{2^2}{5} + \frac{3^2}{7}) = 1 + \frac{4-1}{3} + \frac{9-4}{5} - \frac{3^2}{7} = 3 - \frac{3^2}{7}.

Quando tivermos 10 parcelas: (\frac{1^2}{1} + \frac{2^2}{3} + \frac{3^2}{5} + \frac{4^2}{7} + \frac{5^2}{9} + \frac{6^2}{11} + \frac{7^2}{13} + \frac{8^2}{15} + \frac{9^2}{17} + \frac{10^2}{19}) - (\frac{1^2}{3} + \frac{2^2}{5} + \frac{3^2}{7} + \frac{4^2}{9} + \frac{5^2}{11} + \frac{6^2}{13} + \frac{7^2}{15} + \frac{8^2}{17} + \frac{9^2}{19} + \frac{10^2}{21})

= 1 + \frac{4-1}{3} + \frac{9-4}{5} + \frac{16-9}{7} + \frac{25-16}{9} + \frac{36-25}{11} + \frac{49-36}{13} + \frac{64-49}{15} + \frac{81-64}{17} + \frac{100-81}{19} - \frac{10^2}{21}

= 10 - \frac{10^2}{21}

Assim, quando tivermos n parcelas: (\frac{1^2}{1} + \frac{2^2}{3} + ... + \frac{n^2}{2n-1}) - (\frac{1^2}{3} + \frac{2^2}{5} + ... + \frac{n^2}{2n+1}) = n - \frac{n^2}{2n+ 1}

No caso, 2n+1 = 2003 \Rightarrow n = 1001, logo o número inteiro mais próximo é 1001, alternativa A.
Futuro MATEMÁTICO
e^{\pi \cdot i} +1 = 0
MarceloFantini
Colaborador Moderador
Colaborador Moderador
 
Mensagens: 3126
Registrado em: Seg Dez 14, 2009 11:41
Formação Escolar: GRADUAÇÃO
Andamento: formado

Re: questoes de concurso

Mensagempor Molina » Ter Jul 13, 2010 16:55

Boa tarde, Karla.

Problema interessante. Se depois puder nos dizer de qual concurso foi, ficaria grato.

Vou reescrever os dados que você passou:

a= \frac{1^2}{1}+ \frac{2^2}{3}+\frac{3^2}{5}+....+\frac{1001^2}{2001}

b= \frac{1^2}{3}+\frac{2^2}{5}+\frac{3^2}{7}+....+\frac{1001^2}{2003}

Só que agora vou colocar todas as frações de a e b com o mesmo denominador uma embaixo da outra, para facilitar na visualização da subtração:

a= \frac{1^2}{1}+ \frac{2^2}{3}+\frac{3^2}{5}+\frac{4^2}{7}+....+\frac{1001^2}{2001}

b= \frac{0^2}{1}+ \frac{1^2}{3}+\frac{2^2}{5}+\frac{3^2}{7}+....+\frac{1000^2}{2001}+\frac{1001^2}{2003}

Note que eu não fiz nenhuma alteração, apenas adicionei um termo nulo em b para ficar alinhado os denominadores. Subtraindo a-b, termo a termo com o mesmo denominador você irá obter:

a-b=1+\frac{3}{3}+\frac{5}{5}+\frac{7}{7}+\frac{9}{9}+...+\frac{2001}{2001}-\frac{1001^2}{2003}

a-b=1+1+1+1+...+1-\frac{1001^2}{2003}

a-b=1001-\frac{1001^2}{2003} \approx 500,749875 \approx 501

Bom estudo! :y:
Diego Molina | CV | FB | .COM
Equipe AjudaMatemática.com


"Existem 10 tipos de pessoas: as que conhecem o sistema binário e as que não conhecem."
Avatar do usuário
Molina
Colaborador Moderador - Professor
Colaborador Moderador - Professor
 
Mensagens: 1551
Registrado em: Dom Jun 01, 2008 14:10
Formação Escolar: GRADUAÇÃO
Área/Curso: Licenciatura em Matemática - UFSC
Andamento: formado

Re: questoes de concurso

Mensagempor MarceloFantini » Ter Jul 13, 2010 16:57

Quero pedir desculpas. Errei na hora do resultado final, aqui está o certo: n = 1001 \Rightarrow 1001 - \frac{1001^2}{2003} = 1001 - 500.2501 = 500.7499 , ou seja, o número inteiro mais próximo é 501, letra E.
Futuro MATEMÁTICO
e^{\pi \cdot i} +1 = 0
MarceloFantini
Colaborador Moderador
Colaborador Moderador
 
Mensagens: 3126
Registrado em: Seg Dez 14, 2009 11:41
Formação Escolar: GRADUAÇÃO
Andamento: formado


Voltar para Desafios Médios

 



  • Tópicos relacionados
    Respostas
    Exibições
    Última mensagem

Quem está online

Usuários navegando neste fórum: Nenhum usuário registrado e 1 visitante

 



Assunto: Conjunto dos números racionais.
Autor: scggomes - Sex Fev 18, 2011 10:38

Olá ! Tenho essa dúvida e não consigo montar o problema para resolução:

Qual é o racional não nulo cujo o quadrado é igual à sua terça parte ?

Grata.


Assunto: Conjunto dos números racionais.
Autor: MarceloFantini - Sex Fev 18, 2011 12:27

x^2 = \frac{x}{3}


Assunto: Conjunto dos números racionais.
Autor: scggomes - Sex Fev 18, 2011 12:55

também pensei que fosse assim, mas a resposta é \frac{1}{3}.

Obrigada Fantini.


Assunto: Conjunto dos números racionais.
Autor: MarceloFantini - Sex Fev 18, 2011 13:01

x^2 = \frac{x}{3} \Rightarrow x^2 - \frac{x}{3} = 0 \Rightarrow x \left(x - \frac{1}{3} \right) = 0

Como x \neq 0:

x - \frac{1}{3} = 0 \Rightarrow x = \frac{1}{3}

O que você fez?


Assunto: Conjunto dos números racionais.
Autor: scggomes - Sex Fev 18, 2011 16:17

eu só consegui fazer a igualdade, não consegui desenvolver o restante, não pensei em fatoração, mas agora entendi o que vc fez.

Obrigada.